Bob Knodel and the laser ring gyroscope

  • 194 Replies
  • 42266 Views
*

sandokhan

  • Flat Earth Sultan
  • Flat Earth Scientist
  • 7049
Re: Bob Knodel and the laser ring gyroscope
« Reply #150 on: March 11, 2019, 10:13:01 PM »
The area loop around the equator (a vector) is parallel to the axis of rotation - the axis of earth's rotation so this dot product is maximised.

But for a loop flat on the surface at the equator, its area is normal to the axis of rotation so the dot product is zero.


Once you mention the word AREA you are dealing with the CORIOLIS EFFECT.

Not with the SAGNAC EFFECT.

Moreover you quoted from Neil Ashby who clearly stated these facts.

In a rare admission, even N. Ashby states that the Coriolis force is responsible for the term commonly used in GPS technology for the Sagnac effect:





*

sandokhan

  • Flat Earth Sultan
  • Flat Earth Scientist
  • 7049
Re: Bob Knodel and the laser ring gyroscope
« Reply #151 on: March 11, 2019, 10:19:17 PM »
Your derivation leads to this formula:

dt = 4ωA/c^2

This is the CORIOLIS EFFECT formula.

Here is a direct derivation of the same formula using only the Coriolis force:

https://www.ias.ac.in/article/fulltext/pram/087/05/0071

The derivation has NO LOOPS at all.

Just a comparison of two sides.


Don't even bother bringing up studies/papers which utilise phase conjugate mirrors as that has no relevance to a interferometer without them.

Please update your knowledge on the subject.



page 152 of the pdf document, section Recent Advances in Photorefractive Nonlinear Optics page 4

The MPPC acts like a normal mirror and Sagnac interferometry is obtained.

It is saying the Sagnac effect can be explained as the Coriolis effect, i.e. they are the same


They cannot be the same.

Here is the formula derived by Dr. Silberstein, clearly described as being explained by the Coriolis force:

dt = 4ωA/c^2



Here is my formula:

Here is the derivation of my formula, using TWO LOOPS:

https://www.theflatearthsociety.org/forum/index.php?topic=30499.msg2117351#msg2117351

Here is the final formula:

2(V1L1 + V2L2)/c2

My formula is confirmed at the highest possible scientific level, having been published in the best OPTICS journal in the world, Journal of Optics Letters, and it is used by the US NAVAL RESEARCH OFFICE, Physics Division.

A second reference which confirms my global/generalized Sagnac effect formula.

https://apps.dtic.mil/dtic/tr/fulltext/u2/a206219.pdf

Studies of phase-conjugate optical devices concepts

US OF NAVAL RESEARCH, Physics Division

Dr. P. Yeh
PhD, Caltech, Nonlinear Optics
Principal Scientist of the Optics Department at Rockwell International Science Center
Professor, UCSB
"Engineer of the Year," at Rockwell Science Center
Leonardo da Vinci Award in 1985
Fellow of the Optical Society of America, the Institute of Electrical and Electronics Engineers

page 152 of the pdf document, section Recent Advances in Photorefractive Nonlinear Optics page 4

The MPPC acts like a normal mirror and Sagnac interferometry is obtained.



Phase-Conjugate Multimode Fiber Gyro

Published in the Journal of Optics Letters, vol. 12, page 1023, 1987

page 69 of the pdf document, page 1 of the article


A second confirmation of the fact that my formula is correct.

Here is the first confirmation:



Self-pumped phase-conjugate fiber-optic gyro, I. McMichael, P. Yeh, Optics Letters 11(10):686-8 · November 1986 

http://www.dtic.mil/dtic/tr/fulltext/u2/a170203.pdf (appendix 5.1)


Exactly the formula obtained by Professor Yeh:

φ = -2(φ2 - φ1) = 4π(R1L1 + R2L2)Ω/λc = 4π(V1L1 + V2L2)/λc

Since Δφ = 2πc/λ x Δt, Δt = 2(R1L1 + R2L2)Ω/c2 = 2(V1L1 + V2L2)/c2

CORRECT SAGNAC FORMULA:

2(V1L1 + V2L2)/c2

The very same formula obtained for a Sagnac interferometer which features two different lengths and two different velocities.

http://www.dtic.mil/dtic/tr/fulltext/u2/a170203.pdf

ANNUAL TECHNICAL REPORT PREPARED FOR THE US OF NAVAL RESEARCH.

Page 18 of the pdf document, Section 3.0 Progress:

Our first objective was to demonstrate that the phase-conjugate fiberoptic gyro (PCFOG) described in Section 2.3 is sensitive to rotation. This phase shift plays an important role in the detection of the Sagnac phase shift due to rotation.

Page 38 of the pdf document, page 6 of Appendix 3.1


it does demonstrate the measurement of the Sagnac phase shift Eq. (3)


HERE IS EQUATION (3) OF THE PAPER, PAGE 3 OF APPENDIX 3.1:

φ = -2(φ2 - φ1) = 4π(R1L1 + R2L2)Ω/λc = 4π(V1L1 + V2L2)/λc

Since Δφ = 2πc/λ x Δt, Δt = 2(R1L1 + R2L2)Ω/c2 = 2(V1L1 + V2L2)/c2

CORRECT SAGNAC FORMULA:

2(V1L1 + V2L2)/c2




The Coriolis effect is a physical effect upon the light beams: it is proportional to the area of the interferometer. It is a comparison of two sides.

The Sagnac effect is an electromagnetic effect upon the velocities of the light beams: it is proportional to the radius of rotation. It is a comparison of two loops.

Two different phenomena require two very different formulas.


My SAGNAC EFFECT formula proven and experimentally fully established at the highest possible level of science.

*

JackBlack

  • 21558
Re: Bob Knodel and the laser ring gyroscope
« Reply #152 on: March 11, 2019, 10:22:57 PM »
[irrelevant nonsense removed]

Again, can you show any problem with my derivation?
Can you provide a valid citation for a rotating ring interferometer with normal mirrors which agrees with you, not me?
Can you provide your own derivation, starting from the basics of a stationary loop?
Can you explain why you are claiming that a square interferometer moving uniformly (i.e. all of it moving together with no rotation) will record a shift?

Asserting I am deriving the wrong formula, when that is the subject of debate, is basically just the same as asserting you are right. IT DOES NOTHING TO HELP YOUR CASE.
Bringing up phase conjugate mirrors does not help your case.

*

sandokhan

  • Flat Earth Sultan
  • Flat Earth Scientist
  • 7049
Re: Bob Knodel and the laser ring gyroscope
« Reply #153 on: March 11, 2019, 10:36:55 PM »
Your derivation leads to this formula:

dt = 4ωA/c^2

This is the CORIOLIS EFFECT formula.

Here is a direct derivation of the same formula using only the Coriolis force:

https://www.ias.ac.in/article/fulltext/pram/087/05/0071

The derivation has NO LOOPS at all.

Just a comparison of two sides.

Here is a step by step explanation of how your formula does not make use of LOOPS, which are required in the SAGNAC EFFECT:

https://www.theflatearthsociety.org/forum/index.php?topic=79637.msg2148651#msg2148651


Don't even bother bringing up studies/papers which utilise phase conjugate mirrors as that has no relevance to a interferometer without them.

Please update your knowledge on the subject.



page 152 of the pdf document, section Recent Advances in Photorefractive Nonlinear Optics page 4

The MPPC acts like a normal mirror and Sagnac interferometry is obtained.


Here is the explanation for the stationary loop:

https://www.theflatearthsociety.org/forum/index.php?topic=30499.msg2153966#msg2153966


The uniform/translational/linear Sagnac is a fact of science, look up Professor Ruyong Wang's seminal paper on the subject.


Or use the paper provided by your tag team partner.




Bringing up phase conjugate mirrors does not help your case.

Please update your knowledge on the subject.



page 152 of the pdf document, section Recent Advances in Photorefractive Nonlinear Optics page 4

The MPPC acts like a normal mirror and Sagnac interferometry is obtained.

Here is my formula:

Here is the derivation of my formula, using TWO LOOPS:

https://www.theflatearthsociety.org/forum/index.php?topic=30499.msg2117351#msg2117351

Here is the final formula:

2(V1L1 + V2L2)/c2

My formula is confirmed at the highest possible scientific level, having been published in the best OPTICS journal in the world, Journal of Optics Letters, and it is used by the US NAVAL RESEARCH OFFICE, Physics Division.

A second reference which confirms my global/generalized Sagnac effect formula.

https://apps.dtic.mil/dtic/tr/fulltext/u2/a206219.pdf

Studies of phase-conjugate optical devices concepts

US OF NAVAL RESEARCH, Physics Division

Dr. P. Yeh
PhD, Caltech, Nonlinear Optics
Principal Scientist of the Optics Department at Rockwell International Science Center
Professor, UCSB
"Engineer of the Year," at Rockwell Science Center
Leonardo da Vinci Award in 1985
Fellow of the Optical Society of America, the Institute of Electrical and Electronics Engineers

page 152 of the pdf document, section Recent Advances in Photorefractive Nonlinear Optics page 4

The MPPC acts like a normal mirror and Sagnac interferometry is obtained.



Phase-Conjugate Multimode Fiber Gyro

Published in the Journal of Optics Letters, vol. 12, page 1023, 1987

page 69 of the pdf document, page 1 of the article


A second confirmation of the fact that my formula is correct.

Here is the first confirmation:



Self-pumped phase-conjugate fiber-optic gyro, I. McMichael, P. Yeh, Optics Letters 11(10):686-8 · November 1986 

http://www.dtic.mil/dtic/tr/fulltext/u2/a170203.pdf (appendix 5.1)


Exactly the formula obtained by Professor Yeh:

φ = -2(φ2 - φ1) = 4π(R1L1 + R2L2)Ω/λc = 4π(V1L1 + V2L2)/λc

Since Δφ = 2πc/λ x Δt, Δt = 2(R1L1 + R2L2)Ω/c2 = 2(V1L1 + V2L2)/c2

CORRECT SAGNAC FORMULA:

2(V1L1 + V2L2)/c2

The very same formula obtained for a Sagnac interferometer which features two different lengths and two different velocities.

http://www.dtic.mil/dtic/tr/fulltext/u2/a170203.pdf

ANNUAL TECHNICAL REPORT PREPARED FOR THE US OF NAVAL RESEARCH.

Page 18 of the pdf document, Section 3.0 Progress:

Our first objective was to demonstrate that the phase-conjugate fiberoptic gyro (PCFOG) described in Section 2.3 is sensitive to rotation. This phase shift plays an important role in the detection of the Sagnac phase shift due to rotation.

Page 38 of the pdf document, page 6 of Appendix 3.1


it does demonstrate the measurement of the Sagnac phase shift Eq. (3)


HERE IS EQUATION (3) OF THE PAPER, PAGE 3 OF APPENDIX 3.1:

φ = -2(φ2 - φ1) = 4π(R1L1 + R2L2)Ω/λc = 4π(V1L1 + V2L2)/λc

Since Δφ = 2πc/λ x Δt, Δt = 2(R1L1 + R2L2)Ω/c2 = 2(V1L1 + V2L2)/c2

CORRECT SAGNAC FORMULA:

2(V1L1 + V2L2)/c2




The Coriolis effect is a physical effect upon the light beams: it is proportional to the area of the interferometer. It is a comparison of two sides.

The Sagnac effect is an electromagnetic effect upon the velocities of the light beams: it is proportional to the radius of rotation. It is a comparison of two loops.

Two different phenomena require two very different formulas.


My SAGNAC EFFECT formula proven and experimentally fully established at the highest possible level of science.

*

rabinoz

  • 26528
  • Real Earth Believer
Re: Bob Knodel and the laser ring gyroscope
« Reply #154 on: March 11, 2019, 11:32:50 PM »
Once you mention the word AREA you are dealing with the CORIOLIS EFFECT.
Incorrect! Every expression for the Sagnac effect has the loop area either explicitly or implicitly.

I asked YOU to explain why your Sagnac equation gives a delay when flat on the surface of the equator - you have no done so!

Everyone except you gives the Sagnac delay as proportional to the area of the loop and don't try claiming "Phase-conjugate fiber-optic gyro by Pochi Yeh et al".
Even Pochi Yeh's "Phase-conjugate fiber-optic gyroPhase-conjugate fiber-optic gyro", .

And here's the frequency shift for a large ring laser gyroscope:
, where:
A = enclosed area,
P = perimeter (beam path length),
λ = optical wavelength,
n = normal vector to A and
Ω = rotation vector.

*

Zaphod

  • 137
Re: Bob Knodel and the laser ring gyroscope
« Reply #155 on: March 12, 2019, 12:03:18 AM »
Can you please have this discussion about the sagnac effect elsewhere? Not that it hasn't been done to death on multiple threads before!

*

sandokhan

  • Flat Earth Sultan
  • Flat Earth Scientist
  • 7049
Re: Bob Knodel and the laser ring gyroscope
« Reply #156 on: March 12, 2019, 03:12:16 AM »
Every expression for the Sagnac effect has the loop area either explicitly or implicitly.

There is no area whatsoever involved in the SAGNAC EFFECT formula.

Ever.

The area comes up ONLY in the CORIOLIS EFFECT formula.

Please update your knowledge on the topological considerations of the Sagnac effect:

https://www.theflatearthsociety.org/forum/index.php?topic=30499.msg2023979#msg2023979

The SAGNAC EFFECT is a comparison of two loops, no area comes up at all in the derivation.

The CORIOLIS EFFECT is a comparison of two sides, the area always comes up in the derivation.

A huge difference.

Everyone except you gives the Sagnac delay as proportional to the area of the loop and don't try claiming "Phase-conjugate fiber-optic gyro by Pochi Yeh et al".
Even Pochi Yeh's "Phase-conjugate fiber-optic gyroPhase-conjugate fiber-optic gyro", .


Professor Yeh's SAGNAC LOOP has no area at all.

It is the SAGNAC EFFECT without an area.



NO AREA, just TWO LOOPS.

So, you have just lied, again, to your readers, because you are really desperate.

This is the CORIOLIS EFFECT frequency formula:



It does include the AREA.

This is the SAGNAC EFFECT frequency formula:

Δf = Δφ x c/P = [4L(v1 + v2)]/λP

No area at all, JUST THE VELOCITIES.

For the GINGERino RLG at Gran Sasso, Italy, the SAGNAC EFFECT FREQUENCY is 2,616,666.666 times greater than the Coriolis effect frequency:

https://www.theflatearthsociety.org/forum/index.php?topic=30499.msg2153966#msg2153966 (two consecutive messages)


*

rabinoz

  • 26528
  • Real Earth Believer
Re: Bob Knodel and the laser ring gyroscope
« Reply #157 on: March 12, 2019, 03:33:44 AM »
Every expression for the Sagnac effect has the loop area either explicitly or implicitly.
There is no area whatsoever involved in the SAGNAC EFFECT formula.
Forget it! I'm not interested in your repeatedly posting your totally incorrect Sagnac formula.
As far as I can see, you are the only one that claims that.
Read again: Bob Knodel and the laser ring gyroscope « Reply #155 on: Today at 05:32:50 PM ».
Now do not bother replying in this thread!
If you must clutter things up with more copy-pasta please start a new thread on: "There is no area whatsoever involved in the SAGNAC EFFECT formula".

*

sandokhan

  • Flat Earth Sultan
  • Flat Earth Scientist
  • 7049
Re: Bob Knodel and the laser ring gyroscope
« Reply #158 on: March 12, 2019, 03:47:36 AM »
My formula is being used by the US NAVAL RESEARCH OFFICE, Physics Division.

My formula was peer-reviewed at the highest possible scientific level, in the best OPTICS journal in the world: Journal of Optics Letters.

Here is my formula:

2(V1L1 + V2L2)/c2

Professor Yeh has published over 400 papers in the best scientfic journals in the world, has thirty US patents and has published five textbooks in optics and photonics.

Dr. P. Yeh
PhD, Caltech, Nonlinear Optics
Principal Scientist of the Optics Department at Rockwell International Science Center
Professor, UCSB
"Engineer of the Year," at Rockwell Science Center
Leonardo da Vinci Award in 1985
Fellow of the Optical Society of America, the Institute of Electrical and Electronics Engineers

A second reference which confirms my global/generalized Sagnac effect formula.

https://apps.dtic.mil/dtic/tr/fulltext/u2/a206219.pdf

Studies of phase-conjugate optical devices concepts

US OF NAVAL RESEARCH, Physics Division

page 152 of the pdf document, section Recent Advances in Photorefractive Nonlinear Optics page 4

The MPPC acts like a normal mirror and Sagnac interferometry is obtained.



Phase-Conjugate Multimode Fiber Gyro

Published in the Journal of Optics Letters, vol. 12, page 1023, 1987

page 69 of the pdf document, page 1 of the article


A second confirmation of the fact that my formula is correct.

Here is the first confirmation:



Self-pumped phase-conjugate fiber-optic gyro, I. McMichael, P. Yeh, Optics Letters 11(10):686-8 · November 1986 

http://www.dtic.mil/dtic/tr/fulltext/u2/a170203.pdf (appendix 5.1)


Exactly the formula obtained by Professor Yeh:

φ = -2(φ2 - φ1) = 4π(R1L1 + R2L2)Ω/λc = 4π(V1L1 + V2L2)/λc

Since Δφ = 2πc/λ x Δt, Δt = 2(R1L1 + R2L2)Ω/c2 = 2(V1L1 + V2L2)/c2

CORRECT SAGNAC FORMULA:

2(V1L1 + V2L2)/c2

The very same formula obtained for a Sagnac interferometer which features two different lengths and two different velocities.

http://www.dtic.mil/dtic/tr/fulltext/u2/a170203.pdf

ANNUAL TECHNICAL REPORT PREPARED FOR THE US OF NAVAL RESEARCH.

Page 18 of the pdf document, Section 3.0 Progress:

Our first objective was to demonstrate that the phase-conjugate fiberoptic gyro (PCFOG) described in Section 2.3 is sensitive to rotation. This phase shift plays an important role in the detection of the Sagnac phase shift due to rotation.

Page 38 of the pdf document, page 6 of Appendix 3.1


it does demonstrate the measurement of the Sagnac phase shift Eq. (3)


HERE IS EQUATION (3) OF THE PAPER, PAGE 3 OF APPENDIX 3.1:

φ = -2(φ2 - φ1) = 4π(R1L1 + R2L2)Ω/λc = 4π(V1L1 + V2L2)/λc

Since Δφ = 2πc/λ x Δt, Δt = 2(R1L1 + R2L2)Ω/c2 = 2(V1L1 + V2L2)/c2

CORRECT SAGNAC FORMULA:

2(V1L1 + V2L2)/c2




*

rabinoz

  • 26528
  • Real Earth Believer
Re: Bob Knodel and the laser ring gyroscope
« Reply #159 on: March 12, 2019, 03:52:30 AM »
<<  Make another thread, thank you, Mr Sandokhan! >>

Re: Bob Knodel and the laser ring gyroscope
« Reply #160 on: March 12, 2019, 11:37:00 AM »
<<  Make another thread, thank you, Mr Sandokhan! >>

Yeah, I don't get the point behind the massive walls of formula when the poster won't engage in simple questions like "What is the axis of aether rotation at different latitudes" and "Would a mechanical flywheel gyro and a laser ring gyro give the same rotational readings if in the same position and orientation on the earth."

Is there a complete disconnect between theory and the real world?

I for one would love to know if Bob Knodel's gyro gave the highest angular rate reading when aligned  with its sensitive axis to Polaris.

Re: Bob Knodel and the laser ring gyroscope
« Reply #161 on: March 12, 2019, 11:48:22 AM »
Sando is actually a bot designed to spit out formulas to learn what they are.
Thats why he doesnt really respond.
Sort of like when you sign up for an online service and they req you to type the text shown in a picture.

*

JackBlack

  • 21558
Re: Bob Knodel and the laser ring gyroscope
« Reply #162 on: March 12, 2019, 02:01:28 PM »
Your derivation leads to this formula:
dt = 4ωA/c^2
This is the CORIOLIS EFFECT formula.
Yes, the correct formula which is more commonly known as the Sagnac effect formula, as they are one in the same.
Repeatedly showing that I have gotten the correct formula will not help your case.

You have been completely unable to show a single problem with my formula. Instead all you have been able to do is repeatedly show it is the correct formula.
Just like you have been completely unable to show the Sagnac effect and the Coriolis effect on a counterpropagating light beams is different.

My derivation does use loops. You have seen that every time.
So you lying and saying it doesn't doesn't help your case.
If anyone's doesn't use loops it would be yours as you aren't bothering to determine the time taken to go around the loop and instead are just looking at 2 arms and finding the difference there, and then adding differences up.

So if you want to complain about not using loops, you are complaining about your own derivation.

Now either show what is wrong with my formula, provide a valid citation discussing the Sagnac effect which uses only a simple ring interferometer with normal mirrors with the entire apparatus rotating, or show your own derivation starting from the basics of how long it takes light to propagate around a stationary loop.
If you are unwilling or unable to do that then go away as you are clearly not interested in debate and just want to preach.

*

sandokhan

  • Flat Earth Sultan
  • Flat Earth Scientist
  • 7049
Re: Bob Knodel and the laser ring gyroscope
« Reply #163 on: March 12, 2019, 02:10:10 PM »
Trolling the upper forums won't help you anymore.

Here is a step by step explanation of how your formula does not make use of LOOPS, which are required in the SAGNAC EFFECT:

https://www.theflatearthsociety.org/forum/index.php?topic=79637.msg2148651#msg2148651

And again:

https://www.theflatearthsociety.org/forum/index.php?topic=79637.msg2148867#msg2148867

And again:

https://www.theflatearthsociety.org/forum/index.php?topic=79637.msg2148008#msg2148008

And yet again:

https://www.theflatearthsociety.org/forum/index.php?topic=79637.msg2148569#msg2148569


Your derivation leads to this formula:

dt = 4ωA/c^2

This is the CORIOLIS EFFECT formula.

Here is a direct derivation of the same formula using only the Coriolis force:

https://www.ias.ac.in/article/fulltext/pram/087/05/0071

The derivation has NO LOOPS at all.

Just a comparison of two sides.


Please update your knowledge on the subject.



page 152 of the pdf document, section Recent Advances in Photorefractive Nonlinear Optics page 4

The MPPC acts like a normal mirror and Sagnac interferometry is obtained.


Here is my formula:

Here is the derivation of my formula, using TWO LOOPS:

https://www.theflatearthsociety.org/forum/index.php?topic=30499.msg2117351#msg2117351

Here is the final formula:

2(V1L1 + V2L2)/c2

My formula is confirmed at the highest possible scientific level, having been published in the best OPTICS journal in the world, Journal of Optics Letters, and it is used by the US NAVAL RESEARCH OFFICE, Physics Division.

A second reference which confirms my global/generalized Sagnac effect formula.

https://apps.dtic.mil/dtic/tr/fulltext/u2/a206219.pdf

Studies of phase-conjugate optical devices concepts

US OF NAVAL RESEARCH, Physics Division

Dr. P. Yeh
PhD, Caltech, Nonlinear Optics
Principal Scientist of the Optics Department at Rockwell International Science Center
Professor, UCSB
"Engineer of the Year," at Rockwell Science Center
Leonardo da Vinci Award in 1985
Fellow of the Optical Society of America, the Institute of Electrical and Electronics Engineers

page 152 of the pdf document, section Recent Advances in Photorefractive Nonlinear Optics page 4

The MPPC acts like a normal mirror and Sagnac interferometry is obtained.



Phase-Conjugate Multimode Fiber Gyro

Published in the Journal of Optics Letters, vol. 12, page 1023, 1987

page 69 of the pdf document, page 1 of the article


A second confirmation of the fact that my formula is correct.

Here is the first confirmation:



Self-pumped phase-conjugate fiber-optic gyro, I. McMichael, P. Yeh, Optics Letters 11(10):686-8 · November 1986 

http://www.dtic.mil/dtic/tr/fulltext/u2/a170203.pdf (appendix 5.1)


Exactly the formula obtained by Professor Yeh:

φ = -2(φ2 - φ1) = 4π(R1L1 + R2L2)Ω/λc = 4π(V1L1 + V2L2)/λc

Since Δφ = 2πc/λ x Δt, Δt = 2(R1L1 + R2L2)Ω/c2 = 2(V1L1 + V2L2)/c2

CORRECT SAGNAC FORMULA:

2(V1L1 + V2L2)/c2

The very same formula obtained for a Sagnac interferometer which features two different lengths and two different velocities.

http://www.dtic.mil/dtic/tr/fulltext/u2/a170203.pdf

ANNUAL TECHNICAL REPORT PREPARED FOR THE US OF NAVAL RESEARCH.

Page 18 of the pdf document, Section 3.0 Progress:

Our first objective was to demonstrate that the phase-conjugate fiberoptic gyro (PCFOG) described in Section 2.3 is sensitive to rotation. This phase shift plays an important role in the detection of the Sagnac phase shift due to rotation.

Page 38 of the pdf document, page 6 of Appendix 3.1


it does demonstrate the measurement of the Sagnac phase shift Eq. (3)


HERE IS EQUATION (3) OF THE PAPER, PAGE 3 OF APPENDIX 3.1:

φ = -2(φ2 - φ1) = 4π(R1L1 + R2L2)Ω/λc = 4π(V1L1 + V2L2)/λc

Since Δφ = 2πc/λ x Δt, Δt = 2(R1L1 + R2L2)Ω/c2 = 2(V1L1 + V2L2)/c2

CORRECT SAGNAC FORMULA:

2(V1L1 + V2L2)/c2




The Coriolis effect is a physical effect upon the light beams: it is proportional to the area of the interferometer. It is a comparison of two sides.

The Sagnac effect is an electromagnetic effect upon the velocities of the light beams: it is proportional to the radius of rotation. It is a comparison of two loops.

Two different phenomena require two very different formulas.


My SAGNAC EFFECT formula proven and experimentally fully established at the highest possible level of science.

*

JackBlack

  • 21558
Re: Bob Knodel and the laser ring gyroscope
« Reply #164 on: March 12, 2019, 03:55:08 PM »
Trolling the upper forums won't help you anymore.
You are the one trolling here, and it has never helped you.
Again, can you show what is wrong with my derivation, which clearly uses loops? Just repeating the same assertion doesn't magically mean it doesn't
Can you provide your own derivation starting from the basics of how long it takes light to propagate around a stationary loop?
Can you provide a paper which shows the Sagnac effect for a simple ring interferometer with the entire setup rotating (so no PCMs or the like and no FOCs or the like) have a formula other than what I have (or equivalent to it)?

Until you do, every post you make is just trolling and spamming and wasting everyone's time.

*

rabinoz

  • 26528
  • Real Earth Believer
Re: Bob Knodel and the laser ring gyroscope
« Reply #165 on: March 12, 2019, 04:02:51 PM »
<< If you want to discuss the details of the Sagnac effect make your own thread! >>
Trolling the upper forums won't help you anymore.

*

sandokhan

  • Flat Earth Sultan
  • Flat Earth Scientist
  • 7049
Re: Bob Knodel and the laser ring gyroscope
« Reply #166 on: March 12, 2019, 10:25:51 PM »
The US NAVAL RESEARCH OFFICE, Physics Division, is using my formula.

This formula has been peer reviewed in the Journal of Optics Letters, one of the best scientific journals in the world.

Here is the global/generalized SAGNAC EFFECT formula:

2(V1L1 + V2L2)/c2

https://apps.dtic.mil/dtic/tr/fulltext/u2/a206219.pdf

Studies of phase-conjugate optical devices concepts

US OF NAVAL RESEARCH, Physics Division

Dr. P. Yeh
PhD, Caltech, Nonlinear Optics
Principal Scientist of the Optics Department at Rockwell International Science Center
Professor, UCSB
"Engineer of the Year," at Rockwell Science Center
Leonardo da Vinci Award in 1985
Fellow of the Optical Society of America, the Institute of Electrical and Electronics Engineers

Author of 400 papers on advanced optics, thirty US patents and five textbooks on optics and photonics.



Phase-Conjugate Multimode Fiber Gyro

Published in the Journal of Optics Letters, vol. 12, page 1023, 1987

page 69 of the pdf document, page 1 of the article


The MPPC acts like a normal mirror and Sagnac interferometry is obtained.

page 152 of the pdf document, section Recent Advances in Photorefractive Nonlinear Optics page 4

Self-pumped phase-conjugate fiber-optic gyro, I. McMichael, P. Yeh, Optics Letters 11(10):686-8 · November 1986 

http://www.dtic.mil/dtic/tr/fulltext/u2/a170203.pdf (appendix 5.1)


Exactly the formula obtained by Professor Yeh:

φ = -2(φ2 - φ1) = 4π(R1L1 + R2L2)Ω/λc = 4π(V1L1 + V2L2)/λc

Since Δφ = 2πc/λ x Δt, Δt = 2(R1L1 + R2L2)Ω/c2 = 2(V1L1 + V2L2)/c2

CORRECT SAGNAC FORMULA:

2(V1L1 + V2L2)/c2

The very same formula obtained for a Sagnac interferometer which features two different lengths and two different velocities.

http://www.dtic.mil/dtic/tr/fulltext/u2/a170203.pdf

ANNUAL TECHNICAL REPORT PREPARED FOR THE US OF NAVAL RESEARCH.

Page 18 of the pdf document, Section 3.0 Progress:

Our first objective was to demonstrate that the phase-conjugate fiberoptic gyro (PCFOG) described in Section 2.3 is sensitive to rotation. This phase shift plays an important role in the detection of the Sagnac phase shift due to rotation.

Page 38 of the pdf document, page 6 of Appendix 3.1


it does demonstrate the measurement of the Sagnac phase shift Eq. (3)


HERE IS EQUATION (3) OF THE PAPER, PAGE 3 OF APPENDIX 3.1:

φ = -2(φ2 - φ1) = 4π(R1L1 + R2L2)Ω/λc = 4π(V1L1 + V2L2)/λc

Since Δφ = 2πc/λ x Δt, Δt = 2(R1L1 + R2L2)Ω/c2 = 2(V1L1 + V2L2)/c2

CORRECT SAGNAC FORMULA:

2(V1L1 + V2L2)/c2




Here is a step by step explanation of how your formula does not make use of LOOPS, which are required in the SAGNAC EFFECT:

https://www.theflatearthsociety.org/forum/index.php?topic=79637.msg2148651#msg2148651

And again:

https://www.theflatearthsociety.org/forum/index.php?topic=79637.msg2148867#msg2148867

And again:

https://www.theflatearthsociety.org/forum/index.php?topic=79637.msg2148008#msg2148008

And yet again:

https://www.theflatearthsociety.org/forum/index.php?topic=79637.msg2148569#msg2148569


Your derivation leads to this formula:

dt = 4ωA/c^2

This is the CORIOLIS EFFECT formula.

Here is a direct derivation of the same formula using only the Coriolis force:

https://www.ias.ac.in/article/fulltext/pram/087/05/0071

The derivation has NO LOOPS at all.

Just a comparison of two sides.


Here is the derivation of my formula, which is being used by the US NAVAL RESEARCH OFFICE, using two LOOPS, as required by the correct definition of the SAGNAC EFFECT:

https://www.theflatearthsociety.org/forum/index.php?topic=30499.msg2117351#msg2117351

Here is the final formula:

2(V1L1 + V2L2)/c2

My formula is confirmed at the highest possible scientific level, having been published in the best OPTICS journal in the world, Journal of Optics Letters, and it is used by the US NAVAL RESEARCH OFFICE, Physics Division.

« Last Edit: March 12, 2019, 10:34:06 PM by sandokhan »

*

Zaphod

  • 137
Re: Bob Knodel and the laser ring gyroscope
« Reply #167 on: March 12, 2019, 11:19:50 PM »
Can you PLEASE start a separate thread for the Sagnac merry-go-round! It's most frustrating to have multiple threads derailed by this.

*

rabinoz

  • 26528
  • Real Earth Believer
Re: Bob Knodel and the laser ring gyroscope
« Reply #168 on: March 12, 2019, 11:22:32 PM »
<< If you want to discuss the details of the Sagnac effect make your own thread! >>
Stop spamming this thread! It's about "Re: Bob Knodel and the laser ring gyroscope" not Sandokhan theories on the Sagnac effect.
Of you want any other response Make your own thread!

*

sandokhan

  • Flat Earth Sultan
  • Flat Earth Scientist
  • 7049
Re: Bob Knodel and the laser ring gyroscope
« Reply #169 on: March 13, 2019, 12:20:48 AM »
BK's RLG registered a 15 degree per hour drift.

The RE on youtube are mocking his experiment, saying all the while "I told you so".

Albert Michelson was the first physicist to claim that the formula used in detecting rotation, dt = 4ωA/c^2, is the SAGNAC EFFECT formula.

However, this is the CORIOLIS EFFECT formula, here is a very easy derivation:

https://www.ias.ac.in/article/fulltext/pram/087/05/0071

The derivation has NO LOOPS at all.

The definition of the SAGNAC EFFECT involves TWO LOOPS.

No loops, no SAGNAC EFFECT.

The error committed by Michelson is a fundamental one: he used the Coriolis effect formula to describe a very different phenomenon, the SAGNAC EFFECT.

Here is the derivation of my formula, which is being used by the US NAVAL RESEARCH OFFICE, using two LOOPS, as required by the correct definition of the SAGNAC EFFECT:

https://www.theflatearthsociety.org/forum/index.php?topic=30499.msg2117351#msg2117351

Here is the final formula:

2(V1L1 + V2L2)/c2

My formula is confirmed at the highest possible scientific level, having been published in the best OPTICS journal in the world, Journal of Optics Letters, and it is used by the US NAVAL RESEARCH OFFICE, Physics Division:



https://apps.dtic.mil/dtic/tr/fulltext/u2/a206219.pdf

Studies of phase-conjugate optical devices concepts

US OF NAVAL RESEARCH, Physics Division

Phase-Conjugate Multimode Fiber Gyro

Published in the Journal of Optics Letters, vol. 12, page 1023, 1987

page 69 of the pdf document, page 1 of the article


The MPPC acts like a normal mirror and Sagnac interferometry is obtained.

page 152 of the pdf document, section Recent Advances in Photorefractive Nonlinear Optics page 4

Here is another reference explicitly using my GLOBAL/GENERALIZED SAGNAC EFFECT FORMULA:

Self-pumped phase-conjugate fiber-optic gyro, I. McMichael, P. Yeh, Optics Letters 11(10):686-8 · November 1986 

http://www.dtic.mil/dtic/tr/fulltext/u2/a170203.pdf (appendix 5.1)


Someone has to inform BK that he recorded the Coriolis effect of the ether drift, and that his RLG did not record the SAGNAC EFFECT.


*

JackBlack

  • 21558
Re: Bob Knodel and the laser ring gyroscope
« Reply #170 on: March 13, 2019, 01:00:08 AM »
The US NAVAL RESEARCH OFFICE, Physics Division, is using my formula.
Stop lying.
They are using a different formula, for a completely different setup.

That setup is irrelevant to the discussion at hand as it uses a PCM.

Laser ring gyroscopes, like that used by Bob Knodel which proved the Earth rotates (as do all experiments like this) did not use a PCM.

You have brought up the Sagnac effect and your wild claims about it in countless threads.
You have been refuted in every one of them in that I have seen which allowed debate.
You have refused to engage in any form of rational discussion on the subject and instead just repeatedly assert the same refuted nonsense and dismiss anything that goes against it, even from your own sources.

You are clearly unwilling to debate the Sagnac effect and instead just want to preach.
If you want to preach, do it in the preaching forum, not this one.

And like I have said repeatedly, if you want to appeal to authority, then you've already lost, as all the authorities you are appealing to accept Earth is round and rotating.

*

sandokhan

  • Flat Earth Sultan
  • Flat Earth Scientist
  • 7049
Re: Bob Knodel and the laser ring gyroscope
« Reply #171 on: March 13, 2019, 01:18:58 AM »
The phase-conjugate mirror ACTS AS A NORMAL MIRROR.

Please read.



The MPPC acts like a normal mirror and Sagnac interferometry is obtained.

You are unable to read and comprehend a very simple known scientific fact: the PCM acts as a NORMAL MIRROR.


When faced with the truth, you are accusing someone else of lying, instead of realizing that by now the readers of this thread and of your messages are beginning to realize that you are unable to face reality.


PLEASE READ AGAIN:


The MPPC acts like a normal mirror and Sagnac interferometry is obtained.


The set-up is exactly the same.


That is why I included the exact quotes from the scientific papers, so that you will have no excuse at all in denying this fact.


http://www.dtic.mil/dtic/tr/fulltext/u2/a170203.pdf

ANNUAL TECHNICAL REPORT PREPARED FOR THE US OF NAVAL RESEARCH.

Page 18 of the pdf document, Section 3.0 Progress:

Our first objective was to demonstrate that the phase-conjugate fiberoptic gyro (PCFOG) described in Section 2.3 is sensitive to rotation. This phase shift plays an important role in the detection of the Sagnac phase shift due to rotation.

Page 38 of the pdf document, page 6 of Appendix 3.1


it does demonstrate the measurement of the Sagnac phase shift Eq. (3)



HERE IS EQUATION (3) OF THE PAPER, PAGE 3 OF APPENDIX 3.1:

φ = -2(φ2 - φ1) = 4π(R1L1 + R2L2)Ω/λc = 4π(V1L1 + V2L2)/λc

Since Δφ = 2πc/λ x Δt, Δt = 2(R1L1 + R2L2)Ω/c2 = 2(V1L1 + V2L2)/c2

CORRECT SAGNAC FORMULA:

2(V1L1 + V2L2)/c2



Here is the derivation of my formula, which is being used by the US NAVAL RESEARCH OFFICE, using two LOOPS, as required by the correct definition of the SAGNAC EFFECT:

https://www.theflatearthsociety.org/forum/index.php?topic=30499.msg2117351#msg2117351

Here is the final formula:

2(V1L1 + V2L2)/c2

My formula is confirmed at the highest possible scientific level, having been published in the best OPTICS journal in the world, Journal of Optics Letters, and it is used by the US NAVAL RESEARCH OFFICE, Physics Division.



Your derivation leads to this formula:

dt = 4ωA/c^2

This is the CORIOLIS EFFECT formula.

Here is a direct derivation of the same formula using only the Coriolis force:

https://www.ias.ac.in/article/fulltext/pram/087/05/0071

The derivation has NO LOOPS at all.

Just a comparison of two sides.



Here is the complete refutation of your failed claims:

Here is a step by step explanation of how your formula does not make use of LOOPS, which are required in the SAGNAC EFFECT:

https://www.theflatearthsociety.org/forum/index.php?topic=79637.msg2148651#msg2148651

And again:

https://www.theflatearthsociety.org/forum/index.php?topic=79637.msg2148867#msg2148867

And again:

https://www.theflatearthsociety.org/forum/index.php?topic=79637.msg2148008#msg2148008

And yet again:

https://www.theflatearthsociety.org/forum/index.php?topic=79637.msg2148569#msg2148569




Phase-Conjugate Multimode Fiber Gyro

Published in the Journal of Optics Letters, vol. 12, page 1023, 1987

page 69 of the pdf document, page 1 of the article


THE VERY SAME FORMULA DERIVED BY ME.


*

rabinoz

  • 26528
  • Real Earth Believer
Re: Bob Knodel and the laser ring gyroscope
« Reply #172 on: March 13, 2019, 01:27:52 AM »
<< If you want to discuss the details of the Sagnac effect make your own thread! >>
Stop spamming this thread! It's about "Re: Bob Knodel and the laser ring gyroscope" not Sandokhan theories on the Sagnac effect.
Of you want any other response Make your own thread!

*

rabinoz

  • 26528
  • Real Earth Believer
Re: Bob Knodel and the laser ring gyroscope
« Reply #173 on: March 13, 2019, 02:01:38 AM »
BK's RLG registered a 15 degree per hour drift.
That's his words and the words of others of the Globebusters YouTube site.

Quote from: sandokhan
The RE on youtube are mocking his experiment, saying all the while "I told you so".
The most vicious attacks come from other flat earthers, especially his own Globebusters group. Take a look at:

#TheRealGlobebusters #TEAMYAHAWASHI #FlatEarth
Bob Knodel of Globebusters is a LIAR and a SATANIST and getting THE BOOT by Awake Souls


Quote from: sandokhan
Someone has to inform BK that he recorded the Coriolis effect of the ether drift, and that his RLG did not record the SAGNAC EFFECT.
Well, it looks like you are the someone! No flat earther would take any notice of me on YouTube ::). I'm not exactly flavour-of-month ::).

By the way, do you know who wrote this, "Standard fiber-optic gyros are Sagnac interferometers that are inherently insensitive to reciprocal phase changes and sensitive to nonreciprocal phase changes"?

*

sandokhan

  • Flat Earth Sultan
  • Flat Earth Scientist
  • 7049
Re: Bob Knodel and the laser ring gyroscope
« Reply #174 on: March 13, 2019, 02:30:06 AM »
BK registered the Coriolis effect of the ether drift. This is a basic fact which all RE and FE on youtube must understand.

By the way, do you know who wrote this, "Standard fiber-optic gyros are Sagnac interferometers that are inherently insensitive to reciprocal phase changes and sensitive to nonreciprocal phase changes"?

Is this supposed to be a joke on your part?

You mean you don't know the difference between reciprocal phase changes and nonreciprocal phase changes?

ROTATION (SAGNAC EFFECT) is a nonreciprocal phase change, which produces a net phase shift.

A nonreciprocal phase shift, in an interferometer, is due to the Sagnac effect ( the phase shift produced by the Sagnac effect is nonreciprocal).

*

JackBlack

  • 21558
Re: Bob Knodel and the laser ring gyroscope
« Reply #175 on: March 13, 2019, 02:38:52 AM »
[more garbage preaching]
Like I said, if all you want to do is preach, do it in the believers only section.

If you want to try debating the Sagnac effect, go back to any of the countless threads where you have your ass repeatedly handed to you and discuss it there.

So far all your claims have been refuted and you have been completely unable to show any error from others, and you can't even manage an extremely simple derivation.

This means that the Sagnac effect remains proportional to the area and angular velocity, and that means Bob measured the rotation of the Earth.

*

rabinoz

  • 26528
  • Real Earth Believer
Re: Bob Knodel and the laser ring gyroscope
« Reply #176 on: March 13, 2019, 02:55:09 AM »
BK registered the Coriolis effect of the ether drift. This is a basic fact which all RE and FE on youtube must understand.

By the way, do you know who wrote this, "Standard fiber-optic gyros are Sagnac interferometers that are inherently insensitive to reciprocal phase changes and sensitive to nonreciprocal phase changes"?
Is this supposed to be a joke on your part?
No.

Quote from: sandokhan
ROTATION (SAGNAC EFFECT) is a nonreciprocal phase change, which produces a net phase shift.

A nonreciprocal phase shift, in an interferometer, is due to the Sagnac effect ( the phase shift produced by the Sagnac effect is nonreciprocal).
Answer the question! Do you know who wrote this, "Standard fiber-optic gyros are Sagnac interferometers that are inherently insensitive to reciprocal phase changes and sensitive to nonreciprocal phase changes"?

Now have you contacted Bob Knodel? Remember that the most vicious attacks come from other flat earthers, especially his own Globebusters group. Take a look at:

#TheRealGlobebusters #TEAMYAHAWASHI #FlatEarth
Bob Knodel of Globebusters is a LIAR and a SATANIST and getting THE BOOT by Awake Souls


Quote from: sandokhan
Someone has to inform BK that he recorded the Coriolis effect of the ether drift, and that his RLG did not record the SAGNAC EFFECT.
Well, it looks like you are the someone! No flat earther would take any notice of me on YouTube ::). I'm not exactly flavour-of-month ::).

*

sandokhan

  • Flat Earth Sultan
  • Flat Earth Scientist
  • 7049
Re: Bob Knodel and the laser ring gyroscope
« Reply #177 on: March 13, 2019, 03:08:20 AM »
This means that the Sagnac effect remains proportional to the area and angular velocity

This means that the Sagnac effect remains proportional to the area and angular velocity

Now, there is no escape possible for you from this statement.

SAGNAC EFFECT WITHOUT AN AREA



The most ingenious experiment performed by Professor Yeh: light from a laser is split into two separate fibers, F1 and F2 which are coiled such that light travels clockwise in F1 and counterclockwise in F2.

https://www.researchgate.net/publication/26797550_Self-pumped_phase-conjugate_fiber-optic_gyro

Self-pumped phase-conjugate fiber-optic gyro, I. McMichael, P. Yeh, Optics Letters 11(10):686-8 · November 1986 

http://www.dtic.mil/dtic/tr/fulltext/u2/a170203.pdf (appendix 5.1)


NO AREA INVOLVED IN THIS SAGNAC EXPERIMENT.


Here is a SECOND experiment performed by Professor Yeh, a SAGNAC EXPERIMENT WITHOUT AN AREA:




Phase-conjugate fiber-optic gyro, P. Yeh, I. McMichael, M. Khoshnevisan, Applied Optics 25(7):1029-30 · April 1986

http://www.dtic.mil/dtic/tr/fulltext/u2/a170203.pdf (appendix 5.4)

FINAL FORMULA:

φ = -2(φ2 - φ1) = 4π(R1L1 + R2L2)Ω/λc = 4π(V1L1 + V2L2)/λc

Since Δφ = 2πc/λ x Δt, Δt = 2(R1L1 + R2L2)Ω/c2 = 2(V1L1 + V2L2)/c2

CORRECT SAGNAC FORMULA:

2(V1L1 + V2L2)/c2


ONLY the CORIOLIS EFFECT formula involves an area:

4Aω/c2



Well, it looks like you are the someone! No flat earther would take any notice of me on YouTube

My comments on two different youtube videos, relating to the BK experiment, were deleted within half an hour.

So, if someone here wants to contact BK, or even awakesouls, that would be great.

*

rabinoz

  • 26528
  • Real Earth Believer
Re: Bob Knodel and the laser ring gyroscope
« Reply #178 on: March 13, 2019, 03:36:39 AM »
Well, it looks like you are the someone! No flat earther would take any notice of me on YouTube

My comments on two different youtube videos, relating to the BK experiment, were deleted within half an hour.

So, if someone here wants to contact BK, or even awakesouls, that would be great.
Go and do it yourself! I've no interest in resurrecting Bob Knodel or Globebusters! It's not my problem.

*

JackBlack

  • 21558
Re: Bob Knodel and the laser ring gyroscope
« Reply #179 on: March 13, 2019, 04:04:38 AM »
Now, there is no escape possible for you from this statement.
Good, I don't need one. You do.
Now like I said, if you want to debate the Sagnac effect go bring up one of your old threads.
If you want to just preach, go to the believers only section.

If you want to stay here, discuss Bob knodel measuring the rotation of Earth with the laser ring gyroscope.
Anything else is irrelevant.